LSAT and Law School Admissions Forum

Get expert LSAT preparation and law school admissions advice from PowerScore Test Preparation.

 Administrator
PowerScore Staff
  • PowerScore Staff
  • Posts: 8929
  • Joined: Feb 02, 2011
|
#61020
Please post your questions below!
 Tori_Won
  • Posts: 2
  • Joined: Feb 20, 2019
|
#62800
I narrowed my answer choices down to A and C. Is C incorrect because it places every individual item in an exact place while A still lets L and O go to either day? A little clarification on how you broke down this question would be great. Thanks!
 Rachael Wilkenfeld
PowerScore Staff
  • PowerScore Staff
  • Posts: 1392
  • Joined: Dec 15, 2011
|
#62851
Hi Tori,

You are on the right track! Let's start with what we know about the global rules to the game, before we look at the suspension question.

The global rules tell us that M and N have to be together on the same day, J has to be at 1PM on some day, and L and O have to be on different days.

From the L/O rule we can draw an important inference---MN can't be on the same day as J or K, because that would force L/O on the same day. So we know we have one day that is MN and L/O, and a second day that is JK and L/O. Those are not in time order, but just the variables that are on each day in some order.

The question becomes, without the L/O rule, how do we get that split? If J and K have to be together, we have the exact same set up. MN on one day, JK on the other, with L and O split up. That's what answer choice (A) describes.

The problem with answer choice (C) is that it gives a bit too much information. It says that N and O have to be on the same day. This creates a MNO day and a JKL day. That's a different impact than we originally had, where L and O could be on either day. It's too restrictive.

It sounds to me like that's consistent with what you were thinking. Great job!

Hope that helps.
Rachael
 Tori_Won
  • Posts: 2
  • Joined: Feb 20, 2019
|
#62863
Thanks for the clarification, Rachael!
User avatar
 lsatquestions
  • Posts: 66
  • Joined: Nov 08, 2021
|
#96917
Why is D incorrect?
User avatar
 atierney
PowerScore Staff
  • PowerScore Staff
  • Posts: 215
  • Joined: Jul 06, 2021
|
#96951
D is incorrect because it doesn't prevent L and O from lecturing on the same as each other. Specifically there could be JLO block on Thursday with KMN on Friday.

Get the most out of your LSAT Prep Plus subscription.

Analyze and track your performance with our Testing and Analytics Package.